subject
Mathematics, 14.07.2021 04:10 theblackdot16

A magazine currently has 8300 subscribers for its online web version. It is adding members at the rate of R(t) = 140e0.03t
subscribers per month. If the proportion of members who remain subscribed t months from now is
S(t) = eโˆ’0.06t,
how many online subscribers will the magazine have two years from now? (Round your answer to the nearest integer.)

ansver
Answers: 1

Another question on Mathematics

question
Mathematics, 21.06.2019 14:30
Geometry: determine whether segment mn is parallel to segment kl. justify your answer. jm 6 mk 3 jn 8 nl 4
Answers: 3
question
Mathematics, 21.06.2019 22:00
How do you write a paragraph proof to prove that the corresponding angles shown are congruent
Answers: 2
question
Mathematics, 21.06.2019 23:10
Which best describes the function on the graph? direct variation; k = โˆ’2 direct variation; k = -1/2 inverse variation; k = โˆ’2 inverse variation; k = -1/2
Answers: 2
question
Mathematics, 21.06.2019 23:30
In stepmber, daniel and justin opened a business. during their first three months of business, their average monthly profit was $51. if during their first month, they lost $29, and during their second month, they gained $115, what was their profit or loss for their third plzzz i will give you 299 points
Answers: 1
You know the right answer?
A magazine currently has 8300 subscribers for its online web version. It is adding members at the ra...
Questions
question
Mathematics, 17.10.2020 07:01